Minimizing the sum of fractions

Algebra Level 2

y z x + z x y + x y z \large \frac{yz}{x} + \frac{zx}{y} + \frac{xy}{z}

Let x x , y y and z z be positive reals such that x + y + z = 91 x+y+z=91 , find the minimum of the expression above.


The answer is 91.

This section requires Javascript.
You are seeing this because something didn't load right. We suggest you, (a) try refreshing the page, (b) enabling javascript if it is disabled on your browser and, finally, (c) loading the non-javascript version of this page . We're sorry about the hassle.

4 solutions

Minh Nguyễn
Dec 25, 2016

Applying AM-GM inequality , we have:

  • y z x + z x y 2 z \dfrac{yz}{x}+\dfrac{zx}{y}\geq 2z

  • z x y + x y z 2 x \dfrac{zx}{y}+\dfrac{xy}{z}\geq 2x

  • x y z + y z x 2 y \dfrac{xy}{z}+\dfrac{yz}{x}\geq 2y

Therefore, adding these inequalities up gives x y z + y z x + z x y x + y + z = 91 \dfrac{xy}{z}+\dfrac{yz}{x}+\dfrac{zx}{y}\geq x+y+z=\boxed{91} .

I apologize for the argument earlier. My approach seems to be wrong according. I am sorry.

Chew-Seong Cheong - 4 years, 5 months ago

As with all questions about minimum, other than establishing that we have a lower bound, we should also show that this lower bound can indeed be achieved. For example, while it is true that each term is 0 \geq 0 , we do not say that since y z x + z x y + x y z 0 \frac{yz}{x} + \frac{zx}{y} + \frac{xy}{z} \geq 0 , hence the minimum is 0 (because it cannot be achieved).

Calvin Lin Staff - 4 years, 5 months ago

If X,y,z = equals 30,30,31 respectively we will find the results 89.06 less than the minimum limit as your example 91. Your answer is not accurate.

Mahmoud Elnagar - 3 years, 11 months ago

Log in to reply

With those numbers, I get a sum of 91.03.

Can you recheck your calculations?

Calvin Lin Staff - 3 years, 11 months ago
Reynan Henry
Dec 26, 2016

Relevant wiki: Rearrangement Inequality

c y c x y z c y c x y y = x + y + z = 91 \sum_{cyc}\frac{xy}{z}\ge\sum_{cyc} \frac{xy}{y}=x+y+z=91 rearrangement inequality

As with all questions about minimum, other than establishing that we have a lower bound, we should also show that this lower bound can indeed be achieved. For example, while it is true that each term is 0 \geq 0 , we do not say that since y z x + z x y + x y z 0 \frac{yz}{x} + \frac{zx}{y} + \frac{xy}{z} \geq 0 , hence the minimum is 0 (because it cannot be achieved).

Calvin Lin Staff - 4 years, 5 months ago
Chaebum Sheen
Dec 25, 2016

Note that ( a b ) 2 + ( b c ) 2 + ( c a ) 2 0 (a-b)^2+(b-c)^2+(c-a)^2 \ge 0 a 2 + b 2 + c 2 a b + b c + c a \implies a^2+b^2+c^2 \ge ab+bc+ca From this we can discern ( a + b + c ) 2 3 ( a b + b c + c a ) (1) (a+b+c)^2 \ge 3(ab+bc+ca) \tag{1} 3 ( a 2 + b 2 + c 2 ) ( a + b + c ) 2 (2) 3(a^2+b^2+c^2) \ge (a+b+c)^2 \tag{2}

If a = x y z , b = y z x , c = z x y a=\frac{xy}{z}, b=\frac{yz}{x}, c=\frac{zx}{y} , then note that from (1) \text{(1)} and (2) \text{(2)}

( c y c x y z ) 2 3 ( c y c x 2 ) ( x + y + z ) 2 \left( \sum_{cyc} \frac{xy}{z} \right)^2 \ge 3 \left(\sum_{cyc} x^2 \right) \ge (x+y+z)^2

From this, we have that the following inequality holds with equality when x = y = z x=y=z . c y c x y z 91 \sum_{cyc} \frac{xy}{z} \ge 91

As with all questions about minimum, other than establishing that we have a lower bound, we should also show that this lower bound can indeed be achieved. For example, while it is true that each term is 0 \geq 0 , we do not say that since y z x + z x y + x y z 0 \frac{yz}{x} + \frac{zx}{y} + \frac{xy}{z} \geq 0 , hence the minimum is 0 (because it cannot be achieved).

Calvin Lin Staff - 4 years, 5 months ago

Log in to reply

I have done it.

Chaebum Sheen - 4 years, 5 months ago
Jun Arro Estrella
Jan 12, 2017

Set x= y = z

0 pending reports

×

Problem Loading...

Note Loading...

Set Loading...